URGENT Please helpppp,
I will mark brainliest
Geometry A

URGENT Please Helpppp, I Will Mark BrainliestGeometry A

Answers

Answer 1

Answer:

Step-by-step explanation:

<DCE≅<BCA

BC≅CE

AC≅CD

BA≅DE


Related Questions

Kyle can was the car in 30 minutes. Michael can wash the car in 40 minutes. Working together, can they wash the car in less than 16 minutes?

Answers

No, they can not wash the car in less than 16 minutes while working together.

Given, Kyle can wash the car in 30 minutes.

Michael can wash the car in 40 minutes.

Now, we are asked that working together, can they wash the car in less than 16 minutes.

So, Kyle wash the car = 30 min

Kyle can wash (1/30)th part of the car in 1 min.

Michael wash the car = 40 min

Michael can wash (1/40)th part of the car in 1 min.

Kyle and Michael can together wash the ( 1/30 + 1/40 )th part of the car in 1 min.

Kyle and Michael = 1/30 + 1/40

Kyle and Michael = (4 + 3)/120

Kyle and Michael = 7/120

So, Kyle and Michael can together wash the 7/120th part of the car in 1 min.

and both can together wash the car in 120/7 min i.e. 17.14 min

So, working together both can wash the car in 17.14 min.

Hence, No, they can not wash the car in less than 16 minutes while working together.

Learn more about Direct and Inverse Proportion here https://brainly.com/question/1266676

#SPJ1

20 POINTSSS!!! Graph the ellipse.
Name the coordinates of the center, endpoints of major axis, endpoints of minor axis, and foci of the ellipse.

Answers

The coordinates of the center, endpoints of major axis, endpoints of minor axis, and foci of the ellipse. is (2,-1+3√3) & (2,-1 -3√3).

Calculation:-

[tex]\sqrt{1-\frac{a^{2} }{b^{2} } } =\sqrt{1-\frac{9}{36} } =\sqrt{\frac{36-9}{36} }[/tex]

= [tex]\sqrt{\frac{27}{36} } = \frac{\sqrt{3} }{2}[/tex]

=( b,H ± be) = (2, -1 ± 6x [tex]\frac{\sqrt{3} }{2}[/tex])

(2,-1+3√3) & (2,-1 -3√3).

The long axis is called the major axis and the short axis is called the minor axis. Each major axis endpoint is an ellipse vertex, and each minor axis endpoint is an ellipse vertex. The center of the ellipse is the center of both the major and minor axes. The endpoints of the principal axis are called vertices.

The midpoint of focus is the center of the ellipse. The minor axis is a line segment perpendicular to the major axis, passing through the center, and having ellipses at both ends. Ellipse focal points are two reference points that help draw the ellipse. The foci of the ellipse are on the major axis of the ellipse, equidistant from the origin. An ellipse represents the location of a point that is a constant distance from two fixed points.

Learn more about The ellipse here:-https://brainly.com/question/16904744

#SPJ1

Write the slope-intercept form for an equation of a line that is perpendicular to the graph of y=6x - 6 and passes through the x-intercept of that line.

Answers

Answer:

Step-by-step explanation:

ur mom

4x^4 - 3x^3 + 2x^2 - 5x +6 divided by (x-2)what is the remainder of this question

Answers

Answer:

The remainder is 44

Explanations:

The given polynomial is:

[tex]4x^4-3x^3+2x^2-5x\text{ + 6}[/tex]

The function is to be divided by x - 2

The remainder of the division will be calculated using the remainder theorem

The remainder theorem states that " If a function g(x) is divided by x - a, the remainder of the division will be g(a)"

[tex]\text{Let g(x) = 4x}^4-3x^3+2x^2-5x+6[/tex]

The remainder when g(x) is divided by x -2 is g(2)

[tex]\begin{gathered} g(2)=4(2)^4-3(2)^3+2(2)^2-\text{ 5(2) + 6} \\ g(2)\text{ = 4(16) - 3(8) + 2(4) -5(2) + 6} \\ g(2)\text{ = 64 - 24 + 8 - 10 + 6} \\ g(2)\text{ = 44} \end{gathered}[/tex]

Find 8th term for 10,-20,40

Answers

Answer:

-1280

Step-by-step explanation:

So, as we can see just by loooing at this geometric sequence, the factor is -2. Let's double check this.

10 * -2 = -20 * -2 = 40

We can just keep multiplying to find the 8th term!

40 * -2 = -80 * -2 = 160 *-2 = -320 * -2 = 640 * -2 = -1280

Sometimes with a short sequence, it is better to just multiply it out. You could also do it like this, though:

[tex]a_{n}=a_{1} *r^{n-1}[/tex]

Here, r = -2. We are looking for the 8th term, which would be 8

Plug in our values:

[tex]a_{8}=10*-2^{7}[/tex]

[tex]-2^{7}=-128\\[/tex]

[tex]10*-128=-1280[/tex]

That is the same answer we got before. I would suggest doing it mathematically if you have a long sequence (15, 100, 300 terms)

Hope this helped!


The tallest tree in the United States
Coast Redwood in Jedidiah Smith State
Park in California. It is 321 feet tall.
Suppose you are 5 feet 8 inches tall and
cast a shadow that is 2 feet at a certain
time of day. About how long is the tree's
shadow at the same time of day? (convert
5 ft 8 inches to feet)

Answers

x = 113.2941  is the approximate length of the tree's shadow at the same time of a day.

What is a length?

A unit of length is any arbitrarily chosen and widely accepted reference standard for measuring length. The most common units in modern use are metric units, which are used in every country on the planet. In the United States, customary units are also used. British Imperial units are still used in the United Kingdom and other countries for a variety of purposes. SI and non-SI units are subsets of the metric system.

5 feet + 8 inches is equal to 5 feet.

5 feet 8 inches is equal to 5 feet plus (8 inches) * (1 foot 12 inches).

5 feet 8 inches plus (8/12) feet equals 6 feet.

5 feet 8 inches equals 5 feet plus 2/3 feet.

5 feet 8 inches = (5 + (2/3)) feet

5 feet 8 inches = ((15/3) + (2/3)) feet

5 feet 8 inches = ((15+2)/3) feet

5 feet 8 inches = (17/3) feet

5 feet 8 inches = 5.667 feet

A/B = C/D

where

A = height of person

B = length of shadow of person

C = height of tree

D = length of shadow of tree

So we have

A = 17/3

B = 2

C = 321

D = x

A/B = C/D

(17/3)/2 = 321/x

(17/3)*x = 2*321

(17/3)*x = 642

(3/17)*(17/3)*x = (3/17)*642

x = 1926/17 the precise length of the tree's shadow

The approximate length of the tree's shadow is x = 113.2941.

To know more about length, visit:

https://brainly.com/question/8552546

#SPJ1

List the transformations.

Answers

The transformation of the equation [y = 4 - 1/2 √x + 3] is shown on the graph where the curve touched the x-axis on (61, 0).

What is a graph?

To develop a graph is to make a diagram that depicts the relationship between two or more objects.

Make a sequence of bars on graph paper as an example of a graph.

Bar graphs, circle graphs, and line graphs are the three most often used types of graphs.

Different types of data can be displayed using different types of graphs.

So, plotting the equation on the graph as follows:

The equation: y = 4 - 1/2 √x + 3

Plot the equation on the graph:

(Refer to the graph attached below)

The curve touches the x-axis on the coordinate (61, 0).

Therefore, the transformation of the equation [y = 4 - 1/2 √x + 3] is shown on the graph where the curve touched the x-axis on (61, 0).

Know more about the graph here:

https://brainly.com/question/28953186


#SPJ1

The sale price of women's wool coats is $ ____question attached below

Answers

By '70% off' the text means that there is a discount of 70% on the item, in other words we are only paying 30% of the original value of the item. Therefore if we multiply 30% by the value of the item, we will find the price. This is done below:

[tex]\begin{gathered} \text{price}=220\cdot30\text{ \%} \\ \text{price}=220\cdot\frac{30}{100} \\ \text{price}=220\cdot0.3 \\ \text{price}=66 \end{gathered}[/tex]

The price of the with discount coat is $66

>)How many people were surveyed about their favorite pets?A) 46B) 34C36D) 44

Answers

ANswer:

From the bar diagram,

Number of people own rabbit as pet = 4

Number of people own dog as pet = 8

Number of people own cat as pet = 11

Number of people own goldfish as pet = 6

Number of people own hamster as pet = 5

So, the number of people surveyed about their favorite pets equals the sum of number of people own rabbit, dog, cat, goldfish or hamster as pet.

Number of people surveyed about their favorite pets

[tex]\begin{gathered} =4+8+11+6+5 \\ =34 \end{gathered}[/tex]

Hence 34 people surveyed about their favorite pets.

Option B is correct.

Which number is IRRATIONAL?A)V12B)136C)V64D)V144

Answers

An irrational number is a number that cannot be written as a fraction - the decimal goes on forever without repeating. Therefore:

[tex]\sqrt[]{12}=2\sqrt[]{3}=3.46410[/tex]

in this case it cannot be expressed as a fraction and it is decimal, this is the correct option. So,

answer A.

Hello! Need a little help on this functions question. Thanks!

Answers

..

SOLUTION

EXPLANATION;

The range of the graph is:

[tex][-1,\infty)[/tex][tex]The\text{ range of f\lparen x\rparen=}\sqrt[\placeholder{⬚}]{x+1}-3\text{ is \lbrack-3,}\infty)[/tex]

Therefore the difference is;

[tex][-3,-1)[/tex]

The Arnold Inn offers two plans for wedding parties. Under plan A, the inn charges $30 for each person in attendance. Under plan B, the inn charges $1300 plus $20 for each person in excess of the first 25 who attend. For what size parties will plan B cost less? I do not understand how for Plan b: 1300+20(p-25). I do not understand the part p-25

Answers

ANSWER

81 people

EXPLANATION

Let p be the number of people that attend the party.

Under plan A, the inn charges $30 for each person, so the value y of a party for p people is,

[tex]y_A=30x[/tex]

Then, under plan B, the cost is $1300 for a maximum of 25 people - this means that if 1 to 25 people attend the party, the cost is the same, $1300. For each person in excess of the first 25 - this means for 26, 27, 28, etc, the inn charges $20 each. The cost for plan B is,

[tex]y_B=1300+20(p-25)[/tex]

The last part, (p - 25), is the part of the equation that separates the first 25 attendees. This equation works for 25 people or more, but it is okay to solve this problem. Note that for p = 25, the cost for plan A is,

[tex]y_A=30\cdot25=750[/tex]

Which is less than the cost of plan B ($1300).

We have to find for what number of people attending the party, the cost of plan B is less than the cost of plan A,

[tex]y_BThis is,[tex]1300+20(p-25)<30p[/tex]

We have to solve this for p. First, apply the distributive property of multiplication over addition/subtract4ion to the 20,

[tex]\begin{gathered} 1300+20p-20\cdot25<30p \\ 1300+20p-500<30p \end{gathered}[/tex]

Add like terms,

[tex]\begin{gathered} (1300-500)+20p<30p \\ 800+20p<30p \end{gathered}[/tex]

Now, subtract 20p from both sides,

[tex]\begin{gathered} 800+20p-20p<30p-20p \\ 800<10p \end{gathered}[/tex]

And divide both sides by 10,

[tex]\begin{gathered} \frac{800}{10}<\frac{10p}{10} \\ 80

For 80 people, the costs of the plans are,

[tex]\begin{gathered} y_A=30\cdot80=2400 \\ y_B=1300+20(80-25)=1300+20\cdot55=1300+1100=2400 \end{gathered}[/tex]

Both have the same cost. The solution to the inequation was the number of people, p, is more than 80. This means that for 81 people the cost of plan B should be less than the cost of plan A,

[tex]\begin{gathered} y_A=30\cdot81=2430 \\ y_B=1300+20(81-25)=2420 \end{gathered}[/tex]

For 81 people, plan B costs $10 less than plan A.

Please Help! Parallel Lines and Transversals!

Answers

no they are not parallel because 50 and 133 are same side exterior angles and those are supplementary. However 50+133=183 and not 180.

Answer: No they are not parallel

pls help I was taught this concept today and I can't understand or get it right!! Find the measure of the arc or angle indicatedFind the measure of mPSQa) 53°b) 248°c) 72°d) 65°

Answers

Step 1:

Find the value of x

Using secant theorem

[tex]<\text{PQR =}\frac{1}{2}\text{ }\timesStep 2:

If x = 9

Then 6x+2 = 6(9) + 2 = 54+2 = 56 degrees

The measure of arc PQ = 14(9) - 14 = 126 - 14 = 112 degrees

Step 3:

The total angle in a circumference is 360 degrees

Therefore ,

mPQ + mPSQ = 360

mPSQ = 360 - mPQ

mPSQ= 360 - 112 = 248degrees

The answer is option B

start new workings.
What value of x makes the two expressions
below equal? Give your answer as a
decimal.
5x-8
First expression
3x+5
Second expression give ur answer as a decimal maths

Answers

The value of x that makes both expressions equal is 6.5.

What are expressions?An expression, often known as a mathematical expression, is a finite collection of symbols that are well-formed in accordance with context-dependent principles.As an illustration, the phrase x + y is one where x and y are terms with an addition operator in between. There are two sorts of expressions in mathematics: numerical expressions, which only contain numbers, and algebraic expressions, which also include variables.

So, the value of x that makes both the expression equal.

So, we can write the expression as:

5x - 8 = 3x + 5

Now, solve for x as follows:

5x - 8 = 3x + 5

5x - 3x = 5 + 8

2x = 13

x = 13/2

x = 6.5

Therefore, the value of x that makes both expressions equal is 6.5.

Know more about expressions here:

https://brainly.com/question/28934492

#SPJ1

pls help me wi this question

Answers

Answer:

1 block west and 8 blocks north

Step-by-step explanation:

One block east and two blocks north to the coffee shop. Subtract two blocks west from the one block east, and you get one block west. Add the six blocks north to the two blocks north, and you get eight blocks north.

Andrea labels ten cards with the numbers 1 through 10. She places the cards with prime numbers (2,3,5,7) in one box, and places the rest of the cards in another box. If Andrea draws one random card from the box of prime numbers and then one random card from the other box, how many different pairs of numbers are possible outcomes?

Answers

Box with prime numbers = 2, 3, 5, 7

Box with the other numbers = 1, 4,6,8,9, 10

Possible pairs

12 13 15 17 Four pairs

42 43 45 47 Four pairs

62 63 65 67 Four pairs

82 83 85 87 Four pairs

92 9 3 95 97 Four pairs

102 103 105 107 Four pairs

Total numbers of pairs 24 pairs

Evaluate

107% of 700m

Answers

Answer:

834.6m

Step-by-step explanation:

780*1.07=834.6

107% of 700m is 749 meters

The question and the Triangle is in the same image. I'm on point 3

Answers

ANSWER

• Ratio: 2

,

• Lengths of the sides of the new triangle: 6, 6√2

,

• Circles: ,see explanation

,

• Triangle: ,see explanation

EXPLANATION

• The ratio is the quotient between the length of segment DE and, for example, segment AB,

[tex]\frac{DE}{AB}=\frac{6}{3}=2[/tex]

• Now, we have to multiply each side of triangle ABC by 2 to obtain the lengths of the sides of triangle DEF,

[tex]\begin{cases}DF=2BC=2\cdot3=6 \\ \\ EF=2AC=2\cdot\sqrt[]{3^2+3^2}=2\cdot\sqrt[]{2\cdot9}=6\sqrt[]{2}\end{cases}[/tex]

• Next, draw the two circles mentioned,

• The intersection between the two circles is point F. The ,triangle is,

I’m not sure how to solve 3d. College calculus 1

Answers

Step 1

Given;

[tex]f(x)=(x^2+5)^3[/tex]

Required; To simplify

[tex]\frac{f(x)-f(0)}{x},\text{ x}\ne0[/tex]

Step 2

[tex]\frac{(x^2+5)^3-(0^2+5)^3}{x}[/tex][tex]\mleft(a+b\mright)^3=a^3+3a^2b+3ab^2+b^3---(apply\text{ p}\operatorname{erf}ect\text{ cube formula)}[/tex][tex](x^2+5)^3=(x^2)^3+3(x^2)^2(5)+3x^2(5^2)+5^3[/tex][tex](x^2+5)^3=x^6+15x^4+75x^2+125[/tex][tex]\frac{(x^6+15x^4+75x^2+125)-125}{x}[/tex][tex]\begin{gathered} \frac{x^6+15x^4+75x^2+125-125}{x} \\ \frac{x^6+15x^4+75x^2}{x} \\ \frac{f(x)-f(0)}{x}=x^5+15x^3+75x \end{gathered}[/tex]

Hence if we factorize we get;

[tex]x(x^4+15x^2+75)[/tex]

Therefore;

[tex]\frac{f(x)-f(0)}{x}=x(x^4+15x^2+75)[/tex]

Answer:

See below

Step-by-step explanation:

f(x) = ( x^2 + 5)^3      f(0) =  5^3 = 125

(x^2+5)^3 = x^6 + 15x^4 + 75 x^2 + 125

so you have       (x^6 + 15x^4 + 75x^2  + 125 - 125) /x

                   = x^5 + 15x^3 + 75x     =   x ( x^4 + 15x^2 + 75)  

find the coordinates of the ends of each latus rectum and equations of asymptotes.

Answers

For conic section of the form:

[tex](\frac{x^2}{a^2})-(\frac{y^2}{b^2})=1[/tex]

The Ends of the Lactus Rectum is given as:

[tex]L=(ae,\frac{b^2}{a}),L=(ae,\frac{-b^2}{a})[/tex]

The e in the equation above is the Eccentricity of the Hyperbola.

This can be obtained by the formula:

[tex]e=\frac{\sqrt[]{a^2+b^2}}{a}[/tex]

Thus, comparing the standard form of the conic with the given equation, we have:

[tex]\begin{gathered} \frac{(y+8)^2}{16}-\frac{(x-3)^2}{9}=1 \\ \text{This can be further expressed in the form:} \\ \frac{(y+8)^2}{4^2}-\frac{(x-3)^2}{3^2}=1 \\ By\text{ comparing this with:} \\ \frac{x^2}{a^2}-\frac{y^2}{b^2}=1 \\ We\text{ can deduce that:} \\ a=4;b=3 \end{gathered}[/tex]

Then, we need to obtain the value of the Eccentiricity, e.

[tex]\begin{gathered} e=\frac{\sqrt[]{a^2+b^2}}{a} \\ e=\frac{\sqrt[]{4^2+3^2}}{4} \\ e=\frac{\sqrt[]{16+9}}{4} \\ e=\frac{\sqrt[]{25}}{4}=\frac{5}{4} \end{gathered}[/tex]

Hence, the coordinate of the ends of the each lactus rectum is:

[tex]\begin{gathered} L=(ae,\frac{b^2}{a}),L=(ae,\frac{-b^2}{a}_{}) \\ L=(4\times\frac{5}{4},\frac{3^2}{4}),L=(4\times\frac{5}{4},\frac{-3^2}{4}) \\ L=(5,\frac{9}{4}),L=(5,\frac{9}{4}) \end{gathered}[/tex]

A baker can bake 20 cupcakes in two hours, 30 cupcakes in three hours, and 40 cupcakes in four hours. What is the constant rate of change?

Answers

A baker can bake 20 cupcakes in two hours, 30 cupcakes in three hours, and 40 cupcakes in four hours.

so, the rate can be calculated as following:

number of cupcakes over the number of hours

So,

[tex]\begin{gathered} \frac{20}{2}=10 \\ or,\frac{30}{3}=10 \\ or,\frac{40}{4}=10 \end{gathered}[/tex]

Please help now asap !!!

Answers

answer:

6378/2

step-by-step explanation:

dividing the largest number by the smallest number will allow for the greatest quotient. hope that helps!

The population of a mosquito population obeys the law of uninhibited growth.If there are 500 mosquito initially and there are 800 after 1 day. How long is it until there are 7000 mosquito?Round your answer to the nearest tenth.

Answers

The law of uninhibited growth is express as:

[tex]N(t)=N_0e^{kt}[/tex]

where N(t) is the population at time t, N0 is the initial population, k is the growth rate and t is the time. In this case we know that after one day, t=1, the population is 800 and that the initial population was 500; plugging these values and solving for k we have:

[tex]\begin{gathered} 800=500e^k \\ e^k=\frac{800}{500} \\ \ln e^k=\ln\frac{8}{5} \\ k=\ln\frac{8}{5} \end{gathered}[/tex]

Now that we have the growth rate, we know that the population growth in this case can be express as:

[tex]N(t)=500e^{(\ln\frac{8}{5})t}[/tex]

We want to know the time it takes for the population to be 7000, to find it we equate our expression to this value and solve for t:

[tex]\begin{gathered} 7000=500e^{(\ln\frac{8}{5})t} \\ e^{(\ln\frac{8}{5})t}=\frac{7000}{500} \\ \ln e^{(\ln\frac{8}{5})t}=\ln14 \\ (\ln\frac{8}{5})t=\ln14 \\ t=\frac{\ln14}{\ln\frac{8}{5}} \\ t=5.6 \end{gathered}[/tex]

Therefore, it takes 5.6 days for the population to reach 7000 individuals.

if a/ b+1 = 2, what does 2b equal?

Answers

[tex]\frac{a}{b+1}=2[/tex]

In a certain​ country, 9/50 of college freshmen major in chemistry. A community college in a region of the country has a freshman enrollment of approximately 900 students. How many of these freshmen might we project are majoring in chemistry

Answers

AS per the unitary method, the number of freshmen majoring in chemistry are 162.

Unitary method:

Unitary method defied as a process of finding the value of a single unit, and based on the known values.

Given,

In a certain​ country, 9/50 of college freshmen major in chemistry. A community college in a region of the country has a freshman enrollment of approximately 900 students.

Here we need to find the the number of freshmen majoring in chemistry.

As per the concept of unitary method, let us consider x be the number of freshmen majoring in chemistry.

So, we know that, 9/50 of college freshmen major in chemistry.

The total number of students in the community colleges is 900.

Therefore, the number of freshmen in it is calculated as

x = 900 x 9/50

x = 18 x 9

x = 162

Therefore, there are 162 freshmen in chemistry.

To know more about Unitary method here.

https://brainly.com/question/28276953

#SPJ1

Find the equation of line containing given points. Write the equation in slope- intercept form (0,2)(2,-3)

Answers

Answer:

[tex]y=\frac{-5x}{2}\text{ + 2}[/tex]

Explanation:

Here, we want to get the equation of the line

The general equation of a line in slope-intercept form is:

[tex]y\text{ = mx + b}[/tex]

where m is the slope and b is the y-intercept

We can get the equation through the following:

[tex]\frac{y-y_1}{x-x_1}\text{ = }\frac{y_2-y_1}{x_2-x_1}[/tex]

where (x1,y1) is (0,2) and (x2,y2) is (2,-3)

Substituting the values, we have it that:

[tex]\begin{gathered} \frac{y-2}{x-0}\text{ = }\frac{-3-2}{2-0} \\ \\ \frac{y-2}{x}\text{ = }\frac{-5}{2} \\ \\ \left(y-2\right)\text{ = }\frac{-5x}{2} \\ \\ y\text{ = }\frac{-5x}{2}\text{ + 2} \end{gathered}[/tex]

Given 3 and one-tenth times negative 6 times seven-twelfths, determine the product.
eighteen and one sixtieth
negative eighteen and 7 over 120
ten and 17 over 20
negative 10 and 17 over 20
Question 2(Multiple Choice Worth 2 points)
(One-Step Inequalities MC)

Write the statement "the sum of a number and 18.4 is at least −3.8" as an inequality.

−3.8 + b > 18.4
b + 18.4 ≥ −3.8
b + 18.4 ≤ −3.8
−3.8 + b < 18.4
Question 3(Multiple Choice Worth 2 points)
(Adding and Subtracting Rational Numbers MC)

Simplify negative 3 and two-thirds minus 6 and three-fourths.

3 and one-half
negative 10 and one-twelfth
negative 10 and five-twelfths
negative 18 and one-half
Question 4(Multiple Choice Worth 2 points)
(Writing Two-Step Equations MC)

A new gaming chair costs $455.95. You have already saved $155.95 and earn $37.50 each week babysitting. Write and solve an equation to determine how many weeks, w, you must babysit to earn enough money to buy the new gaming chair.

37.5 + 155.95w = 455.95; w = 8
37.5w + 155.95 = 455.95; w = 8
37.5w − 155.95 = 455.95; w = 16
37.5w − 455.95 = 155.95; w = 16
Question 5(Multiple Choice Worth 2 points)
(Dividing Rational Numbers MC)

Divide negative 3 and one-sixth ÷ negative 8 and two-fifths.

252 over 95
95 over 252
51 over 30
30 over 51
Question 6(Multiple Choice Worth 2 points)
(Solving Two-Step Equations MC)

Solve one eighth times quantity x plus 32 end quantity equals negative 7.

x = 56
x = 7
x = −88
x = −24
Question 7(Multiple Choice Worth 2 points)
(Rewriting Rational Numbers LC)

Which number is equal to four and one-sixth?

four and sixteen hundredths with the six repeating
six twenty-fifths
4.2
416%
Question 8(Multiple Choice Worth 2 points)
(Laws of Exponents with Whole Number Exponents LC)

Which expression is equivalent to 2 and one tenth raised to the fifth power divided by nine tenths raised to the fourth power, all raised to the fifth power?

0.55
0.56
2 and one tenth raised to the twenty fifth power divided by nine tenths raised to the twentieth power
2 and one tenth raised to the tenth power divided by nine tenths raised to the ninth power
Question 9(Multiple Choice Worth 2 points)
(Adding and Subtracting Rational Numbers MC)

Julie wanted to match Lisa's obstacle course record of 68.2 seconds. She has already spent thirty-nine and one fourth seconds on wall climbing and 12.84 seconds on the ropes. How much time did she have left to match the record?

12.24 seconds
16.11 seconds
26.41 seconds
28.95 seconds
Question 10(Multiple Choice Worth 2 points)
(Writing Two-Step Equations MC)

Write the math sentence as an equation: Negative nine times the sum of a number and 12.3 is 30.8

−9(r − 12.3) = 30.8
−9r + 12.3 = 30.8
−9r − 12.3 = 30.8
−9(r + 12.3) = 30.8
Question 11(Multiple Choice Worth 2 points)
(Laws of Exponents with Whole Number Exponents MC)

Evaluate two and two tenths raised to the sixth power divided by two and two tenths raised to the fifth power, all raised to the second power.

4.84
4.4
2.2
1
Question 12(Multiple Choice Worth 2 points)
(Adding and Subtracting Linear Expressions MC)

Write the expression in simplest form.

the quantity negative three fifths x minus 8 end quantity minus the quantity negative 14 plus three tenths x end quantity

negative nine tenths x plus 6
negative nine tenths x minus 6
negative three tenths x minus 6
negative three tenths x plus 6
Question 13(Multiple Choice Worth 2 points)
(Order of Operations MC)

Simplify the expression.

negative 15 plus the quantity negative 1 and six tenths plus 9 and 34 hundredths end quantity divided by 6 all times 3 squared minus 5 and 7 tenths

−4.125
−9.09
−12.96
129.09
Question 14(Multiple Choice Worth 2 points)
(Equivalent Linear Expressions LC)

Which expression is equivalent to −4(b − 5)?

−4b − 20
4b + 5
−4b + 20
−4b − 5
Question 15(Multiple Choice Worth 2 points)
(Solving Two-Step Equations MC)

Solve the equation for x.

−0.24x − 16.4 = 1.96

x = −60.2
x = 60.2
x = −76.5
x = 76.5

Answers

To solve the multiple questions performed, a process is being followed to get accurate results which are mentioned after the respective questions.

What are different operations in mathematics?

Addition is the process of joining two or more numbers to determine their total. The result of adding the numbers 3 and 4 is 3 + 4 = 7, for instance. Any sort of number, including whole integers, fractions, decimals, and even negative values, can be added to.

The process of subtracting involves calculating the difference between two numbers. For example, subtracting 4 from 7 yields 7 - 4 = 3. Any sort of number, including whole integers, fractions, decimals, and negative values, can be subtracted.

Multiplication is the process of combining two or more numbers to determine their product. The result of multiplying the numbers 3 and 4 is 3 * 4 = 12, for instance. such as addition. Any kind of number may be used for addition, subtraction, and multiplication.

The act of dividing two integers involves determining their quotient. For example, dividing 12 by 4 yields 12 / 4 = 3. Any sort of value could be divided, although some combinations of numbers, such dividing by zero, are not described as division.

How to solve?

1) 3 * (-6) * (7/12) which is -7.5. So, the answer is -18 and 7/120 or negative 18 and 7 over 120.

2) -3.8 + b - 18.4 ≥ 0

-22.2 + b >= 0

∵sum of a number and 18.4 is at least -3.8, we can say that b is greater than or equal to -22.2.

∴b + 18.4 ≥ −3.8.

3) (-3/3) - (6/4) = (-9/3) - (6/4)

(-9/3) - (6/4) = (-9/3) * (4/4) - (6/4) * (3/3)

= (-36/12) - (18/12)

(-36/12) - (18/12) = -54/12 = -9/2 = -4 and 1/2

The correct answer is negative 18 and one-half.

4) 37.50w = $300

w = 8

37.5w + 155.95 = 455.95; w = 8.

5) (-7/6) / (-17/5) = (-7/6) * (5/17)

= (-35/102)

(-35/102) / (35/35) = (-1/3) / 1

= -1/3

6)x/8 + 32 = -7

x/8 = -39

x = -8 * -39

x = 8 * 39

x = 312

7)x = 56

x = 7

x = −88

x = −24

8)= [(2 and one tenth)^5] / [(9 tenths)^4]^5

= [2^5 * (1/10)^5] / [9^4 * (1/10)^4]^5

= [2^5 * (1/10)^5] / [9^4 * (1/10)^4]^5

= 2^5 * (1/10)^5 / (9^4 * (1/10)^4)^5

= 2^5 * (1/10)^5 / 9^4 * (1/10)^4 * 5

= 2^5 * (1/10)^5 / 9^4 * (1/10)^4 * 5

= 2^5 / 10^5 / 9^4 / 10^4 * 5

= 2^5 / (10^5 * 9^4) / 10^4 * 5

= 2^5 / (10^5 * 9^4) / 10^4 * 5

= 32 / (10^9 * 6561) * 5

= 32 / (10^9 * 6561) * 5

= 32 / 65610000 * 5

= 32 / 65610000 * 5

= 32 / 3285050000

= 32 / 3285050000

= 0.0000977077

To learn more about different operations in mathematics, visit:

https://brainly.com/question/20628271

#SPJ1

the circle graph shows how a family budgets its annual income. if the total annual income is $90,000 what amount is budgeted for clothing?

Answers

Answer:

$11700

Step-by-step explanation:

comment if u need the explanantion ;D

The opposite of a number and sometimes negative true or false?

Answers

The given statement "Negative is the opposite of an integer" is FALSE.

What is an integer?A number that may be expressed without a fractional component is called an integer. For instance, while 9.75, 5+1/2, and 2 are not numbers, 21 and 4 and 2048 are.

So, on the number line, opposite values are those that are equally spaced apart from zero.

Nevertheless, the values' signs will differ.Since both negative and positive numbers can be found in an integer. As a result, an integer's opposite is not always a negative. Take the numbers 3 and 8 as an example.

These numbers' opposites are:

3 is the reciprocal of -3.8's polar opposite is -8.

Therefore, the given statement "Negative is the opposite of an integer" is FALSE.

Know more about integers here:

https://brainly.com/question/28965717

#SPJ1

Other Questions
Select three education policies from the articles above under the control of federal, state, and local government (one federal, one state, one local) . Write a paragraph for each policy explaining why it should, or should not, be controlled at that level. Be sure to explain your reasoning. Use evidence from your lesson and the two articles to support your position. 13 In August, 85% of the middle school students voted in a school election. The number of students who voted was 544.How many students are in the middle school?(A) 462 students(B) 544 students(C)629 students(D) 640 students Which choices are equivalent to the expression below? Check all that apply.A.B.C.72D.E.F. Match the letters with the main components involved in protein synthesis some members of the media often adopt strongly skeptical, even hostile, postures toward the government and elected officials. this is known as what type of journalism? what are the solutions of the equation 0 equals x ^ 2 + 3x - 10 What is molecular genetic manipulation Please explain On a piece of paper, graph y+25**-1. Then determine which answer choicematches the graph you drew.ABD0.9.-3)0,-)(0-3)69,-2)(4-2)(4.23(2)O A. Graph AB. Graph BO C. Graph CO D. Graph D (X^-3y^2/x^3)^-2Simplify the expression. Your final answer should use positive exponents. 1. P, Q and R are three buildings. A car began its journey at P, drove to Q, then to R and returned to P. The bearing of Q from P is 058 and R is due east of Q. PQ = 114 km and QR = 70 km. Draw a clearly labelled diagram to represent the above informationen on the diagram TRund (a) the north/south direction (b) the bearing 058 (c) the distances 114 km and 70 km. (ii) Calculate (a) the measure of angle POR (b) the distance PR [3] (c) the bearing of P from R [3] Find the area of the shaded sector QRS A carpenter cuts a 5-ft board in two pieces. One piece must be three times as longas the other. Find the length of each piece. An account earns an annual rate of 5.4% compounded monthly. If $3,000 is deposited into this account, then after 3 years there is $___. Round your answer to two decimals. Examine Documents A and B. Which of the following statements best explain the racial and social tensions found within the colonial Spanish empire? Choose two correct responses.AThe Creoles felt disrespected by the peninsulars (European-born Spanish) because the latter group held most of the political power within Spanish Latin America.BThe Spanish viewed the Creoles as second class citizens, often assigning them to labor-intensive jobs like their Native America and African slaves.CBecause they were born in the Americas, Creoles had to fight land disputes with Native Americans, while also maintaining their Spanish identity.DThe peninsulars were economically independent, but held no political power. During the eighteen hundreds, Creoles controlled most of the judgeships in Latin America.EThe Spanish treated all citizens within their empire equally. If you have a 77.2% and you got 34% on a test and its worth 60% of your grade, what would you grade be now? Write the expression in the standard form a + bi. How diborane is prepared? write the reaction for the preparation of diborane what digit is in the How to substitute 8x-4y=-12, 8x+y=-17 Collected by the fbi and reported quarterly and annual, the what is considered to be most widely used source of national crime and delinquency staticis in the United States